Seguimos buscando a Arshak. Ayudanos compartiendo!
Encuesta no oficial de docentes
Resultados de la encuesta no oficial de docentes
Probaste el SIGA Helper?

Donar $100 Donar $200 Donar $500 Donar mensualmente


Enviar respuesta 
 
Calificación:
  • 0 votos - 0 Media
  • 1
  • 2
  • 3
  • 4
  • 5
Buscar en el tema
DUDA EN ANALISIS MATEMATICO 2
Autor Mensaje
AGUSTIN27 Sin conexión
Secretario de la SAE
INGENIEROOO :)
******

Ing. en Sistemas
Facultad Regional Buenos Aires

Mensajes: 642
Agradecimientos dados: 20
Agradecimientos: 114 en 25 posts
Registro en: Feb 2010
Mensaje: #1
DUDA EN ANALISIS MATEMATICO 2 Ejercicios Análisis Matemático II
TENGO EL SIGUIENTE INCONVENIENTE:
CALCULE MEDIANTE INTEGRALES TRIPLES EL VOLUMEN DEL CUERPO H, USANDO EL SISTEMA DE COORDENADAS QUE CREA MAS CONVENIENTE.
H ESTA DEFINIDO POR 2Y >= X^2 + Z , X + Y <= 4 , 1º OCTANTE.

SI ALGUIEN ME PODRIA DAR UNA MANO, SE LO AGRADECERIA MUCHO !
18-06-2010 18:54
Encuentra todos sus mensajes Agregar agradecimiento Cita este mensaje en tu respuesta
Saga Sin conexión
Colaborador
out of order
********

Ing. Industrial
Facultad Regional Buenos Aires

Mensajes: 3.768
Agradecimientos dados: 176
Agradecimientos: 1.741 en 931 posts
Registro en: Sep 2009
Mensaje: #2
Re: DUDA EN ANALISIS MATEMATICO 2
Hola, supongo que H esta definido de está manera

\[H: \left\{\begin{matrix} 2y \geq{} x^2+z &\\x+y \leq{} 4 & \end{matrix} \right\] en el primer octante

Una forma de resolverlo, hace un dibujo del problema para poder visualizarlo sino no vas a entender nada thumbdown
Encontra los puntos \[x,y\] imponiendo que \[z=0\], y tomando solo la igualdad entre las funciones, una vez obtenido el punto, simplemente despeja las funciones que te dan, quedaría algo así

\[0 \leq{} x \leq{} 2\]

\[\displaystyle\frac{x^2}{2} \leq{} y \leq{} 4-x\]

\[0 \leq{} z \leq{} 2y-x^2\]

de donde

\[\displaystyle\int \displaystyle\int \displaystyle\int f(x,y,z)dV= \displaystyle\int_{0}^{2} \displaystyle\int_{ \displaystyle\frac{x^2}{2} }^{4-x} \displaystyle\int_{0}^{2y-x^2}dzdydx=\dfrac{68}{5}\]

eso si no me equivoque en las cuentas, =P en cuanto al sistema de coordenadas yo lo dejaría como esta, fijate que no tenes funciones dificiles de integrar todas son sencillas.

saludos wave
19-06-2010 08:54
Encuentra todos sus mensajes Agregar agradecimiento Cita este mensaje en tu respuesta
AGUSTIN27 Sin conexión
Secretario de la SAE
INGENIEROOO :)
******

Ing. en Sistemas
Facultad Regional Buenos Aires

Mensajes: 642
Agradecimientos dados: 20
Agradecimientos: 114 en 25 posts
Registro en: Feb 2010
Mensaje: #3
Re: DUDA EN ANALISIS MATEMATICO 2
Gracias maestro, se hace muy dificil seguir la materia de manera cuatrimestral, y por primera vez, no terminana de dar un concepto que ya tenes que saber el otro, como si fuese tan facil integrar en 3 dimensiones, y lo puedas aprender en unos dias :S .
te hago otra pregunta, si no te jode, acerca de la resolucion de este tipo de ejercicios.

tengo
H: z >= x^2 ; x >= z^2 ; x >= I y I ( módulo de "y" )

puse los limites de integracion asi : -x <= y <= x ; x^2 <= z <= x^(1/2) ( raíz cuadrada de x )

y ahora yo me pregunto, entre que valores hago variar a " x " ???

gracias por la respuesta













aoleonsr escribió:Hola, supongo que H esta definido de está manera

\[H: \left\{\begin{matrix} 2y \geq{} x^2+z &\\x+y \leq{} 4 & \end{matrix} \right\] en el primer octante

Una forma de resolverlo, hace un dibujo del problema para poder visualizarlo sino no vas a entender nada :thumbdown:
Encontra los puntos \[x,y\] imponiendo que \[z=0\], y tomando solo la igualdad entre las funciones, una vez obtenido el punto, simplemente despeja las funciones que te dan, quedaría algo así

\[0 \leq{} x \leq{} 2\]

\[\displaystyle\frac{x^2}{2} \leq{} y \leq{} 4-x\]

\[0 \leq{} z \leq{} 2y-x^2\]

de donde

\[\displaystyle\int \displaystyle\int \displaystyle\int f(x,y,z)dV= \displaystyle\int_{0}^{2} \displaystyle\int_{ \displaystyle\frac{x^2}{2} }^{4-x} \displaystyle\int_{0}^{2y-x^2}dzdydx=\dfrac{68}{5}\]

eso si no me equivoque en las cuentas, :P en cuanto al sistema de coordenadas yo lo dejaría como esta, fijate que no tenes funciones dificiles de integrar todas son sencillas.

saludos :wave:
20-06-2010 19:44
Encuentra todos sus mensajes Agregar agradecimiento Cita este mensaje en tu respuesta
Saga Sin conexión
Colaborador
out of order
********

Ing. Industrial
Facultad Regional Buenos Aires

Mensajes: 3.768
Agradecimientos dados: 176
Agradecimientos: 1.741 en 931 posts
Registro en: Sep 2009
Mensaje: #4
Re: DUDA EN ANALISIS MATEMATICO 2
Hola, supongo que el ejercicio es este

\[H: \left\{z \geq x^2 \wedge x\geq z^2 \wedge x\geq \left |y}\right |\right\}}\]

Sugerencia: dibuja asi lo vas a poder visualizar, sino te sale, ayudate con la pc o un graficador, estos ejercicios los sacas dibujando, observando la proyeccion sobre los planos coordenados, casi siempre se proyectan sobre el xy, pero no se descartan los otros, así podés definir el intervalo sobre el eje x.
Armamos un sistema de ecuaciones tomando solo la igualdad

1) \[x= \left |{y}\right |\]
2) \[z=x^2\]
3) \[x=z^2\]

de 2 y 3 definimos la ecuación

\[x^4-x=0\] las raices evalualas en 1) te queda

\[x=1\rightarrow{\left |{y}\right |}=1\]
\[x=0\rightarrow{\left |{y}\right |}=0\]

ahora de \[z\geq x^2\wedge x \geq z^2\] deducimos

\[x^2\leq z \leq -\sqrt[]{x}\]

\[1\leq x\leq -y\]
,
\[-1\leq y\leq 0\] si \[z<0\] si es \[z\] no me equivocque, explicarlo por acá es medio difícil, eso sale del dibujo del ejercicio

ó

\[x^2\leq z \leq\sqrt[]{x}\]

\[y\leq x\leq 1\]

\[0\leq y\leq 1\] si \[z>0\] podés tomar dos integrales en el intervalo sobre el eje \[y\in{}\left[-1,0\right]\cup\left[0,1\right]\] o simplemente

\[\dfrac{V}{2}\diplaystyle\int \diplaystyle\int \diplaystyle\int f(x,y,z)dV=\]

\[\displaystyle\int_0^1 \displaystyle\int_y^1 \displaystyle\int_{x^2}^{\sqrt[]{x}}dzdxdy=\dfrac{3}{10}\] si no me equivoque en las cuentas, fijate que cambie el orden de los diferenciales ya que así es más

fácil integrarlo de esa manera, para entenderlo mejor te lo vuelvo a repetir, estos ejercicios salen del dibujo, trata de hacerlo, te das cuenta los límites de integracion y del vólumen a integrar, este en

especial es horrible yuk,

Puede haber distintas maneras definir los límites de integración, yo te puse de la manera que lo se hacer, si a alguien por acá se le ocurre otra forma mas corta, avise thumbup3

saludos wave

PD: por tu perfil estas en sistemas ¿ porque cuatri analisis 2? no era anual ?? que desinformado que estoy si cambiaron el programa de sistemas =P rofl zzz
21-06-2010 01:14
Encuentra todos sus mensajes Agregar agradecimiento Cita este mensaje en tu respuesta
AGUSTIN27 Sin conexión
Secretario de la SAE
INGENIEROOO :)
******

Ing. en Sistemas
Facultad Regional Buenos Aires

Mensajes: 642
Agradecimientos dados: 20
Agradecimientos: 114 en 25 posts
Registro en: Feb 2010
Mensaje: #5
Re: DUDA EN ANALISIS MATEMATICO 2
muchas gracias maestro, si yo estoy en sistemas y la curso de forma cuatrimestral.
me podrias decir que programa utilizas para graficar? yo me baje el derive 6.
gracias por tu ayuda che
no entendi igualmente por que dividiste al volumen sobre 2 ?
21-06-2010 14:25
Encuentra todos sus mensajes Agregar agradecimiento Cita este mensaje en tu respuesta
Saga Sin conexión
Colaborador
out of order
********

Ing. Industrial
Facultad Regional Buenos Aires

Mensajes: 3.768
Agradecimientos dados: 176
Agradecimientos: 1.741 en 931 posts
Registro en: Sep 2009
Mensaje: #6
Re: DUDA EN ANALISIS MATEMATICO 2
AGUSTIN27 escribió:muchas gracias maestro, si yo estoy en sistemas y la curso de forma cuatrimestral.
me podrias decir que programa utilizas para graficar? yo me baje el derive 6.
gracias por tu ayuda che

Los dibujo en una hoja, baje un graficador y thumbdown no lo entendi mucho wall asi que lo hago con birome y papel =P aparte en un parcial o final no te dejan tener graficadores, asi que

no es demás practicarlo ;) .

Cita:no entendi igualmente por que dividiste al volumen sobre 2 ?

Fijate que \[y\in{}\left[-1,0\right]\cup\ \left[0,1\right]\] hay simetría respecto del eje x en el vólumen a integrar o sea el

\[V=\displaystyle\int \displaystyle\int \displaystyle\int f(x,y,z)dV+ \displaystyle\int \displaystyle\int \displaystyle\int f(x,y,z)dV=\]

\[=2\left(\displaystyle\int \displaystyle\int \displaystyle\int f(x,y,z)dV\right)\]

que es lo mismo que \[\dfrac{V}{2}=\displaystyle\int \displaystyle\int \displaystyle\int f(x,y,z)dV\] por ahí te confundio que en la resolución anterior se me paso poner el igual antes de la

integral, un error tipográfico wall .

Saludos y suerte en la carrera, si tenés dudas pregunta y se ayuda en lo que se puede ;) wave
21-06-2010 18:31
Encuentra todos sus mensajes Agregar agradecimiento Cita este mensaje en tu respuesta
AGUSTIN27 Sin conexión
Secretario de la SAE
INGENIEROOO :)
******

Ing. en Sistemas
Facultad Regional Buenos Aires

Mensajes: 642
Agradecimientos dados: 20
Agradecimientos: 114 en 25 posts
Registro en: Feb 2010
Mensaje: #7
Re: DUDA EN ANALISIS MATEMATICO 2
Aca vengo con otro, la verdad es que no entiendo el metodo de planteo que se tiene que hacer, intente graficar pero me cuesta una banda comprender el dibujo, por otro lado, ya se como graficar con el derive, no es tan jodido al fin de cuentas.. ( estuve intentando un buen rato jaja )

en fin ..
me dice que calcule el volumen bla bla bla entre .. y >= x^2 , x^2 + y^2 <= 2 , z >= 0 , z <= x ..

segun como me dijiste vos tengo que igualar, es decir:
1) y = x^2
2) x^2 + y^2 = 2
3) z=0
4) z=x

de 1) y 2) concluyo que : x^2 + x^4 = 2 , lo que me queda X=1 y x= -1 ( ambas raices dobles )

y ahora viene la parte que no entiendo, como te das cuenta cuales son los limites de integracion? es decir .. entre que valores varia x e y ?
" z " me quedaria asi : 0 < z < x


gracias por tu buena predisposicion capo, la verdad que se me hace complicadisimo ! tengo el libro de flax, pero no te explica como resolver, si no que resuelve y ya ... tenes algun libro para recomendarme?

muchas gracias ! y saludos
21-06-2010 20:46
Encuentra todos sus mensajes Agregar agradecimiento Cita este mensaje en tu respuesta
Saga Sin conexión
Colaborador
out of order
********

Ing. Industrial
Facultad Regional Buenos Aires

Mensajes: 3.768
Agradecimientos dados: 176
Agradecimientos: 1.741 en 931 posts
Registro en: Sep 2009
Mensaje: #8
Re: DUDA EN ANALISIS MATEMATICO 2
Hola, disculpa por contestar recien hoy , estaba con el tema de la presentacion de unos tps, a ver tu ejercicio es

\[W:\left\{ y\geq{x^2}, x^2+y^2\leq{2}, z\geq{0}, z\leq{x}\right\}\] igualando como hiciste te queda que

\[y=x^2\]

\[x^2+y^2=2\]

de donde para los límites en \[x\] tenemos \[x^2=1\] pero que pasa, si hiciste el dibujo habras notado que descartamos \[x=-1\] entonces \[0\leq x \leq 1\]

El \[y\] lo obtenes de estas ecuaciones

\[x^2+y^2\leq{}2\]

\[y\geq{}x^2\]

de donde obtenés

\[x^2\leq{y}\leq{\sqrt[ ]{2-x^2}}\]

en \[z\] lo tenés bien

\[0\leq{z}\leq{x}\]

la integral queda

\[\displaystyle\int_{0}^{1}\displaystyle\int_{0}^{x}\displaystyle\int_{x^2}^{\sqrt[ ]{2-x^2}}dydzdx=....\] no hice las cuentas por falta de tiempo, fijate si te da el resultado , cualquier duda ;)

saludos
23-06-2010 10:21
Encuentra todos sus mensajes Agregar agradecimiento Cita este mensaje en tu respuesta
AGUSTIN27 Sin conexión
Secretario de la SAE
INGENIEROOO :)
******

Ing. en Sistemas
Facultad Regional Buenos Aires

Mensajes: 642
Agradecimientos dados: 20
Agradecimientos: 114 en 25 posts
Registro en: Feb 2010
Mensaje: #9
Re: DUDA EN ANALISIS MATEMATICO 2
Gracias amigo por la respuesta, pero lo que no entiendo yo ahora es ? porq integras primeros sobre Y , despues sobre Z , despues sobre X ?
de que depende ese cambio de orden ? es decir porque integraste en este orden : dy dz dx , en vez de esta manera dz dy dx ? Creo yo, que no estoy entendiendo eso.
Por otro lado, x= -1 lo descartamos, por que esta como del lado de atras, y no lo tenemos en cuenta? es decir, calculamos solo en el 1er octante ? .

te queria agradecer de verdad por responderme capo, y tomarte tu tiempo en explciarme esto que tanto me cueste. te deseo la mayor de las suertes en lo tuyo !!
23-06-2010 19:30
Encuentra todos sus mensajes Agregar agradecimiento Cita este mensaje en tu respuesta
Saga Sin conexión
Colaborador
out of order
********

Ing. Industrial
Facultad Regional Buenos Aires

Mensajes: 3.768
Agradecimientos dados: 176
Agradecimientos: 1.741 en 931 posts
Registro en: Sep 2009
Mensaje: #10
Re: DUDA EN ANALISIS MATEMATICO 2
Hola

AGUSTIN27 escribió:Gracias amigo por la respuesta, pero lo que no entiendo yo ahora es ? porq integras primeros sobre Y , despues sobre Z , despues sobre X ?
de que depende ese cambio de orden ? es decir porque integraste en este orden : dy dz dx , en vez de esta manera dz dy dx ? Creo yo, que no estoy entendiendo eso.

simplemente lo hice por comodidad en cuentas, fijate que de la manera que vós lo planteas también llegas a lo mismo, la segunda y tercera integral cuyos límites de integración tanto inferior como superior son funciones o constantes , vós las acomodas como mejor veas conveniente, en la primera integral deben quedar si o si los límites de integración tanto inferior como superior iguales a constantes no pueden aparecer funciones en esta integral.
Cita:Por otro lado, x= -1 lo descartamos, por que esta como del lado de atras, y no lo tenemos en cuenta? es decir, calculamos solo en el 1er octante ? .

Vuelvo a repetirte eso sale del dibujo, si haces el dibujo de todas esas superfies verás que el vólumen de dichas intersecciones solo queda en el primer octante, por eso se descarta \[x=-1\], los dibujos son fáciles capo, siempre empeza dibujando sobre el plano xy, por ejemplo para este ejercicio, tenes sobre el plano xy una parabola \[y=x^2\] y una circunferencia \[x^2+y^2=2\] estas son las proyecciones de tu superficie sobre el plano xy, estas serian las directrizes o curvas de nivel de la superficie en \[R^3\] si sobre esas directrizes o curvas de nivel vos haces rectas perpendiculares (generatrices) a ellas, sabes que en \[R^3\] la superficie es un cilindro y un paraboloide de una hoja, y esto solo queda intersectarlo con el plano \[x=z\], bajo las condiciones del ejercicio o sea el mayor igual, menor igual, el volúmen a integrar queda en el primer octante , dibujalo te vas a dar cuenta porque se descarta \[x=-1\], no es complicado por ahi solo un poco de imaginación =P .

saludos, si tenes dudas pss se ayuda hasta donde se pueda thumbup3 wave
23-06-2010 20:18
Encuentra todos sus mensajes Agregar agradecimiento Cita este mensaje en tu respuesta
Buscar en el tema
Enviar respuesta 




Usuario(s) navegando en este tema: 1 invitado(s)